Why don't I get the correct limit of a sequence, regardless of how I arrange it (the sequence), while following the rules for solving limits?Finding the upper and lower limit of the following sequence.limit rules - How to apply limit rules when there are some nonexistent limitsHow to compute the limit of $a_n$ from the following expression?How to evaluate the following limits?How can I get the result of this limitsHow to prove that a limit exists for $lim_limitsnto inftyfracn+2n = 1$Updating the Definition of Limit of a Sequence to take in consideration sequences that are not defined for some finite number of natural numbersFind the formula for the nth term of the sequence and find its limitsHow to solve the sequence limit

What's the biggest organic molecule that could have a smell?

Do all humans have an identical nucleotide sequence for certain proteins, e.g haemoglobin?

Can I say "I have encrypted something" if I hash something?

The case of the pranking snowplow

What is Japanese Language Stack Exchange called in Japanese?

How to read torque specs off this Nissan service diagram?

Why does F + F' = 1?

Insert str into larger str in the most pythonic way

My employer wants me to do a work of 6 months in just 2 months

Can I disable a battery powered device by reversing half of its batteries?

Can a new chain significantly improve the riding experience? If yes - what else can?

Should I leave the first authourship of our paper to the student who did the project whereas I solved it?

How do I politely hint customers to leave my store, without pretending to need leave store myself?

Seized engine due to being run without oil

Do Milankovitch Cycles fully explain climate change?

How does Vivi differ from other Black Mages?

Is there a star over my head?

Do any aircraft carry boats?

Exact Brexit date and consequences

Why was "leaping into the river" a valid trial outcome to prove one's innocence?

My research paper filed as a patent in China by my Chinese supervisor without me as inventor

How seriously should I take a CBP interview where I was told I have a red flag and could only stay for 30 days?

Might have gotten a coworker sick, should I address this?

Gas pipes - why does gas burn "outwards?"



Why don't I get the correct limit of a sequence, regardless of how I arrange it (the sequence), while following the rules for solving limits?


Finding the upper and lower limit of the following sequence.limit rules - How to apply limit rules when there are some nonexistent limitsHow to compute the limit of $a_n$ from the following expression?How to evaluate the following limits?How can I get the result of this limitsHow to prove that a limit exists for $lim_limitsnto inftyfracn+2n = 1$Updating the Definition of Limit of a Sequence to take in consideration sequences that are not defined for some finite number of natural numbersFind the formula for the nth term of the sequence and find its limitsHow to solve the sequence limit






.everyoneloves__top-leaderboard:empty,.everyoneloves__mid-leaderboard:empty,.everyoneloves__bot-mid-leaderboard:empty margin-bottom:0;








2












$begingroup$


The fact that, when solving limits of sequences ($n in mathbbN$), it strikes me as very bizarre that, even though I follow all the (elementary) rules for solving limits (of sequences), I can get different results for the same example! Consider the following:



$$
lim_n to infty left( frac3n + 12n - 1 right) = fraclim_n to infty(3n + 1)lim_n to infty (2n - 1) = fraclim_n to infty(3n) + lim_n to infty(1)lim_n to infty (2n) - lim_n to infty(1) = fracinfty + 1infty - 1 Longrightarrow textThis sequence does not have a limit.
$$



I followed all of the rules for solving limits, yet this conclusion is wrong. The correct approach is the following.



$$
lim_n to infty left( frac3n + 12n - 1 right) = lim_n to infty left( fracn(3 + frac1n)n(2 - frac1n) right) = lim_n to infty left( frac3 + frac1n2 - frac1n right) =fraclim_n to infty(3 + frac1n)lim_n to infty (2 - frac1n) = fraclim_n to infty(3) + lim_n to infty(frac1n)lim_n to infty (2) - lim_n to infty(frac1n) = frac3 + 02 - 0 = frac32.
$$



My question is: why do I get the correct answer only if I arrange the sequence a certain way? I don't understand that at all. Thank you for your answers in advance and God, Jesus Christ, bless and keep you always.










share|cite|improve this question









$endgroup$









  • 6




    $begingroup$
    The error is in the first part: you are assuming that if you get $fracinfty+1infty-1$, then the limit doesn't exist. This is wrong. You cannot conclude anything about the existence of the limit if you get a so-called "indeterminate form," such as $fracinftyinfty$ or $frac00$.
    $endgroup$
    – symplectomorphic
    8 hours ago











  • $begingroup$
    @symplectomorphic Could you elaborate on that, please?
    $endgroup$
    – Gregor Perčič
    8 hours ago






  • 1




    $begingroup$
    Take the limit of x/x as x approaches zero. Obviously the answer is 1, but taking the limit of the top and bottom separately gives 0/0. This is an indeterminate form, not a limit that doesn't exist.
    $endgroup$
    – Gabe
    8 hours ago










  • $begingroup$
    If a limit is of the "form" $fracinftyinfty,$ that does not mean that the limit does not exst.
    $endgroup$
    – Thomas Andrews
    8 hours ago










  • $begingroup$
    Alternatively you can first apply multiplication and addition laws to see that $$ lim_ntoinftyfrac3n+12n−1=frac32lim_ntoinftyfrac6n+26n−3 =frac32+frac32lim_ntoinftyfrac56n−3 $$
    $endgroup$
    – LutzL
    8 hours ago

















2












$begingroup$


The fact that, when solving limits of sequences ($n in mathbbN$), it strikes me as very bizarre that, even though I follow all the (elementary) rules for solving limits (of sequences), I can get different results for the same example! Consider the following:



$$
lim_n to infty left( frac3n + 12n - 1 right) = fraclim_n to infty(3n + 1)lim_n to infty (2n - 1) = fraclim_n to infty(3n) + lim_n to infty(1)lim_n to infty (2n) - lim_n to infty(1) = fracinfty + 1infty - 1 Longrightarrow textThis sequence does not have a limit.
$$



I followed all of the rules for solving limits, yet this conclusion is wrong. The correct approach is the following.



$$
lim_n to infty left( frac3n + 12n - 1 right) = lim_n to infty left( fracn(3 + frac1n)n(2 - frac1n) right) = lim_n to infty left( frac3 + frac1n2 - frac1n right) =fraclim_n to infty(3 + frac1n)lim_n to infty (2 - frac1n) = fraclim_n to infty(3) + lim_n to infty(frac1n)lim_n to infty (2) - lim_n to infty(frac1n) = frac3 + 02 - 0 = frac32.
$$



My question is: why do I get the correct answer only if I arrange the sequence a certain way? I don't understand that at all. Thank you for your answers in advance and God, Jesus Christ, bless and keep you always.










share|cite|improve this question









$endgroup$









  • 6




    $begingroup$
    The error is in the first part: you are assuming that if you get $fracinfty+1infty-1$, then the limit doesn't exist. This is wrong. You cannot conclude anything about the existence of the limit if you get a so-called "indeterminate form," such as $fracinftyinfty$ or $frac00$.
    $endgroup$
    – symplectomorphic
    8 hours ago











  • $begingroup$
    @symplectomorphic Could you elaborate on that, please?
    $endgroup$
    – Gregor Perčič
    8 hours ago






  • 1




    $begingroup$
    Take the limit of x/x as x approaches zero. Obviously the answer is 1, but taking the limit of the top and bottom separately gives 0/0. This is an indeterminate form, not a limit that doesn't exist.
    $endgroup$
    – Gabe
    8 hours ago










  • $begingroup$
    If a limit is of the "form" $fracinftyinfty,$ that does not mean that the limit does not exst.
    $endgroup$
    – Thomas Andrews
    8 hours ago










  • $begingroup$
    Alternatively you can first apply multiplication and addition laws to see that $$ lim_ntoinftyfrac3n+12n−1=frac32lim_ntoinftyfrac6n+26n−3 =frac32+frac32lim_ntoinftyfrac56n−3 $$
    $endgroup$
    – LutzL
    8 hours ago













2












2








2





$begingroup$


The fact that, when solving limits of sequences ($n in mathbbN$), it strikes me as very bizarre that, even though I follow all the (elementary) rules for solving limits (of sequences), I can get different results for the same example! Consider the following:



$$
lim_n to infty left( frac3n + 12n - 1 right) = fraclim_n to infty(3n + 1)lim_n to infty (2n - 1) = fraclim_n to infty(3n) + lim_n to infty(1)lim_n to infty (2n) - lim_n to infty(1) = fracinfty + 1infty - 1 Longrightarrow textThis sequence does not have a limit.
$$



I followed all of the rules for solving limits, yet this conclusion is wrong. The correct approach is the following.



$$
lim_n to infty left( frac3n + 12n - 1 right) = lim_n to infty left( fracn(3 + frac1n)n(2 - frac1n) right) = lim_n to infty left( frac3 + frac1n2 - frac1n right) =fraclim_n to infty(3 + frac1n)lim_n to infty (2 - frac1n) = fraclim_n to infty(3) + lim_n to infty(frac1n)lim_n to infty (2) - lim_n to infty(frac1n) = frac3 + 02 - 0 = frac32.
$$



My question is: why do I get the correct answer only if I arrange the sequence a certain way? I don't understand that at all. Thank you for your answers in advance and God, Jesus Christ, bless and keep you always.










share|cite|improve this question









$endgroup$




The fact that, when solving limits of sequences ($n in mathbbN$), it strikes me as very bizarre that, even though I follow all the (elementary) rules for solving limits (of sequences), I can get different results for the same example! Consider the following:



$$
lim_n to infty left( frac3n + 12n - 1 right) = fraclim_n to infty(3n + 1)lim_n to infty (2n - 1) = fraclim_n to infty(3n) + lim_n to infty(1)lim_n to infty (2n) - lim_n to infty(1) = fracinfty + 1infty - 1 Longrightarrow textThis sequence does not have a limit.
$$



I followed all of the rules for solving limits, yet this conclusion is wrong. The correct approach is the following.



$$
lim_n to infty left( frac3n + 12n - 1 right) = lim_n to infty left( fracn(3 + frac1n)n(2 - frac1n) right) = lim_n to infty left( frac3 + frac1n2 - frac1n right) =fraclim_n to infty(3 + frac1n)lim_n to infty (2 - frac1n) = fraclim_n to infty(3) + lim_n to infty(frac1n)lim_n to infty (2) - lim_n to infty(frac1n) = frac3 + 02 - 0 = frac32.
$$



My question is: why do I get the correct answer only if I arrange the sequence a certain way? I don't understand that at all. Thank you for your answers in advance and God, Jesus Christ, bless and keep you always.







sequences-and-series limits






share|cite|improve this question













share|cite|improve this question











share|cite|improve this question




share|cite|improve this question










asked 8 hours ago









Gregor PerčičGregor Perčič

6691 gold badge5 silver badges12 bronze badges




6691 gold badge5 silver badges12 bronze badges










  • 6




    $begingroup$
    The error is in the first part: you are assuming that if you get $fracinfty+1infty-1$, then the limit doesn't exist. This is wrong. You cannot conclude anything about the existence of the limit if you get a so-called "indeterminate form," such as $fracinftyinfty$ or $frac00$.
    $endgroup$
    – symplectomorphic
    8 hours ago











  • $begingroup$
    @symplectomorphic Could you elaborate on that, please?
    $endgroup$
    – Gregor Perčič
    8 hours ago






  • 1




    $begingroup$
    Take the limit of x/x as x approaches zero. Obviously the answer is 1, but taking the limit of the top and bottom separately gives 0/0. This is an indeterminate form, not a limit that doesn't exist.
    $endgroup$
    – Gabe
    8 hours ago










  • $begingroup$
    If a limit is of the "form" $fracinftyinfty,$ that does not mean that the limit does not exst.
    $endgroup$
    – Thomas Andrews
    8 hours ago










  • $begingroup$
    Alternatively you can first apply multiplication and addition laws to see that $$ lim_ntoinftyfrac3n+12n−1=frac32lim_ntoinftyfrac6n+26n−3 =frac32+frac32lim_ntoinftyfrac56n−3 $$
    $endgroup$
    – LutzL
    8 hours ago












  • 6




    $begingroup$
    The error is in the first part: you are assuming that if you get $fracinfty+1infty-1$, then the limit doesn't exist. This is wrong. You cannot conclude anything about the existence of the limit if you get a so-called "indeterminate form," such as $fracinftyinfty$ or $frac00$.
    $endgroup$
    – symplectomorphic
    8 hours ago











  • $begingroup$
    @symplectomorphic Could you elaborate on that, please?
    $endgroup$
    – Gregor Perčič
    8 hours ago






  • 1




    $begingroup$
    Take the limit of x/x as x approaches zero. Obviously the answer is 1, but taking the limit of the top and bottom separately gives 0/0. This is an indeterminate form, not a limit that doesn't exist.
    $endgroup$
    – Gabe
    8 hours ago










  • $begingroup$
    If a limit is of the "form" $fracinftyinfty,$ that does not mean that the limit does not exst.
    $endgroup$
    – Thomas Andrews
    8 hours ago










  • $begingroup$
    Alternatively you can first apply multiplication and addition laws to see that $$ lim_ntoinftyfrac3n+12n−1=frac32lim_ntoinftyfrac6n+26n−3 =frac32+frac32lim_ntoinftyfrac56n−3 $$
    $endgroup$
    – LutzL
    8 hours ago







6




6




$begingroup$
The error is in the first part: you are assuming that if you get $fracinfty+1infty-1$, then the limit doesn't exist. This is wrong. You cannot conclude anything about the existence of the limit if you get a so-called "indeterminate form," such as $fracinftyinfty$ or $frac00$.
$endgroup$
– symplectomorphic
8 hours ago





$begingroup$
The error is in the first part: you are assuming that if you get $fracinfty+1infty-1$, then the limit doesn't exist. This is wrong. You cannot conclude anything about the existence of the limit if you get a so-called "indeterminate form," such as $fracinftyinfty$ or $frac00$.
$endgroup$
– symplectomorphic
8 hours ago













$begingroup$
@symplectomorphic Could you elaborate on that, please?
$endgroup$
– Gregor Perčič
8 hours ago




$begingroup$
@symplectomorphic Could you elaborate on that, please?
$endgroup$
– Gregor Perčič
8 hours ago




1




1




$begingroup$
Take the limit of x/x as x approaches zero. Obviously the answer is 1, but taking the limit of the top and bottom separately gives 0/0. This is an indeterminate form, not a limit that doesn't exist.
$endgroup$
– Gabe
8 hours ago




$begingroup$
Take the limit of x/x as x approaches zero. Obviously the answer is 1, but taking the limit of the top and bottom separately gives 0/0. This is an indeterminate form, not a limit that doesn't exist.
$endgroup$
– Gabe
8 hours ago












$begingroup$
If a limit is of the "form" $fracinftyinfty,$ that does not mean that the limit does not exst.
$endgroup$
– Thomas Andrews
8 hours ago




$begingroup$
If a limit is of the "form" $fracinftyinfty,$ that does not mean that the limit does not exst.
$endgroup$
– Thomas Andrews
8 hours ago












$begingroup$
Alternatively you can first apply multiplication and addition laws to see that $$ lim_ntoinftyfrac3n+12n−1=frac32lim_ntoinftyfrac6n+26n−3 =frac32+frac32lim_ntoinftyfrac56n−3 $$
$endgroup$
– LutzL
8 hours ago




$begingroup$
Alternatively you can first apply multiplication and addition laws to see that $$ lim_ntoinftyfrac3n+12n−1=frac32lim_ntoinftyfrac6n+26n−3 =frac32+frac32lim_ntoinftyfrac56n−3 $$
$endgroup$
– LutzL
8 hours ago










2 Answers
2






active

oldest

votes


















6














$begingroup$

You did not in fact follow all the rules (and this is a good example of why understanding why the rules are true is important). The relevant rule is:




If both $lim_nrightarrow af(n)$ and $lim_nrightarrow ag(n)$ exist and are finite (and $lim_nrightarrow ag(n)not=0$), then $$lim_nrightarrow af(n)over g(n)=lim_nrightarrow af(n)over lim_nrightarrow ag(n).$$




There are various other rules of similar flavor. But the point is that the "rule" you've tried to apply is not one of them. (And it's a good exercise at this point to go through your textbook and look at what the rules do in fact say, and note that none of them actually get you what you want.)



Indeed, the example you give is a good example of the limitations of these rules: while we can often manipulate limits "algebraically," we can't do this in all cases, and some hypotheses are needed. It's also a good example of why proofs are important, since there are plenty of plausible-sounding statements which are in fact false.






share|cite|improve this answer









$endgroup$














  • $begingroup$
    Hmmm, thank you very much, I didn't know about this caveat! We only scratched the surface of what we can do with limits, as I only studied them in the context of sequences and series. Jesus bless and keep you always. Amen.
    $endgroup$
    – Gregor Perčič
    8 hours ago


















2














$begingroup$

Your reference page here starts with this introduction (emphasis mine):




Listed here are a couple of basic limits and the standard limit laws which, when used in conjunction, can find most limits. They are listed for standard, two-sided limits, but they work for all forms of limits. However, note that if a limit is infinite, then the limit does not exist.




So everywhere below that warning, if there is a statement that a limit must exist, then it's implicit that the limit must not be $infty$.






share|cite|improve this answer









$endgroup$

















    Your Answer








    StackExchange.ready(function()
    var channelOptions =
    tags: "".split(" "),
    id: "69"
    ;
    initTagRenderer("".split(" "), "".split(" "), channelOptions);

    StackExchange.using("externalEditor", function()
    // Have to fire editor after snippets, if snippets enabled
    if (StackExchange.settings.snippets.snippetsEnabled)
    StackExchange.using("snippets", function()
    createEditor();
    );

    else
    createEditor();

    );

    function createEditor()
    StackExchange.prepareEditor(
    heartbeatType: 'answer',
    autoActivateHeartbeat: false,
    convertImagesToLinks: true,
    noModals: true,
    showLowRepImageUploadWarning: true,
    reputationToPostImages: 10,
    bindNavPrevention: true,
    postfix: "",
    imageUploader:
    brandingHtml: "Powered by u003ca class="icon-imgur-white" href="https://imgur.com/"u003eu003c/au003e",
    contentPolicyHtml: "User contributions licensed under u003ca href="https://creativecommons.org/licenses/by-sa/4.0/"u003ecc by-sa 4.0 with attribution requiredu003c/au003e u003ca href="https://stackoverflow.com/legal/content-policy"u003e(content policy)u003c/au003e",
    allowUrls: true
    ,
    noCode: true, onDemand: true,
    discardSelector: ".discard-answer"
    ,immediatelyShowMarkdownHelp:true
    );



    );














    draft saved

    draft discarded
















    StackExchange.ready(
    function ()
    StackExchange.openid.initPostLogin('.new-post-login', 'https%3a%2f%2fmath.stackexchange.com%2fquestions%2f3351115%2fwhy-dont-i-get-the-correct-limit-of-a-sequence-regardless-of-how-i-arrange-it%23new-answer', 'question_page');

    );

    Post as a guest















    Required, but never shown

























    2 Answers
    2






    active

    oldest

    votes








    2 Answers
    2






    active

    oldest

    votes









    active

    oldest

    votes






    active

    oldest

    votes









    6














    $begingroup$

    You did not in fact follow all the rules (and this is a good example of why understanding why the rules are true is important). The relevant rule is:




    If both $lim_nrightarrow af(n)$ and $lim_nrightarrow ag(n)$ exist and are finite (and $lim_nrightarrow ag(n)not=0$), then $$lim_nrightarrow af(n)over g(n)=lim_nrightarrow af(n)over lim_nrightarrow ag(n).$$




    There are various other rules of similar flavor. But the point is that the "rule" you've tried to apply is not one of them. (And it's a good exercise at this point to go through your textbook and look at what the rules do in fact say, and note that none of them actually get you what you want.)



    Indeed, the example you give is a good example of the limitations of these rules: while we can often manipulate limits "algebraically," we can't do this in all cases, and some hypotheses are needed. It's also a good example of why proofs are important, since there are plenty of plausible-sounding statements which are in fact false.






    share|cite|improve this answer









    $endgroup$














    • $begingroup$
      Hmmm, thank you very much, I didn't know about this caveat! We only scratched the surface of what we can do with limits, as I only studied them in the context of sequences and series. Jesus bless and keep you always. Amen.
      $endgroup$
      – Gregor Perčič
      8 hours ago















    6














    $begingroup$

    You did not in fact follow all the rules (and this is a good example of why understanding why the rules are true is important). The relevant rule is:




    If both $lim_nrightarrow af(n)$ and $lim_nrightarrow ag(n)$ exist and are finite (and $lim_nrightarrow ag(n)not=0$), then $$lim_nrightarrow af(n)over g(n)=lim_nrightarrow af(n)over lim_nrightarrow ag(n).$$




    There are various other rules of similar flavor. But the point is that the "rule" you've tried to apply is not one of them. (And it's a good exercise at this point to go through your textbook and look at what the rules do in fact say, and note that none of them actually get you what you want.)



    Indeed, the example you give is a good example of the limitations of these rules: while we can often manipulate limits "algebraically," we can't do this in all cases, and some hypotheses are needed. It's also a good example of why proofs are important, since there are plenty of plausible-sounding statements which are in fact false.






    share|cite|improve this answer









    $endgroup$














    • $begingroup$
      Hmmm, thank you very much, I didn't know about this caveat! We only scratched the surface of what we can do with limits, as I only studied them in the context of sequences and series. Jesus bless and keep you always. Amen.
      $endgroup$
      – Gregor Perčič
      8 hours ago













    6














    6










    6







    $begingroup$

    You did not in fact follow all the rules (and this is a good example of why understanding why the rules are true is important). The relevant rule is:




    If both $lim_nrightarrow af(n)$ and $lim_nrightarrow ag(n)$ exist and are finite (and $lim_nrightarrow ag(n)not=0$), then $$lim_nrightarrow af(n)over g(n)=lim_nrightarrow af(n)over lim_nrightarrow ag(n).$$




    There are various other rules of similar flavor. But the point is that the "rule" you've tried to apply is not one of them. (And it's a good exercise at this point to go through your textbook and look at what the rules do in fact say, and note that none of them actually get you what you want.)



    Indeed, the example you give is a good example of the limitations of these rules: while we can often manipulate limits "algebraically," we can't do this in all cases, and some hypotheses are needed. It's also a good example of why proofs are important, since there are plenty of plausible-sounding statements which are in fact false.






    share|cite|improve this answer









    $endgroup$



    You did not in fact follow all the rules (and this is a good example of why understanding why the rules are true is important). The relevant rule is:




    If both $lim_nrightarrow af(n)$ and $lim_nrightarrow ag(n)$ exist and are finite (and $lim_nrightarrow ag(n)not=0$), then $$lim_nrightarrow af(n)over g(n)=lim_nrightarrow af(n)over lim_nrightarrow ag(n).$$




    There are various other rules of similar flavor. But the point is that the "rule" you've tried to apply is not one of them. (And it's a good exercise at this point to go through your textbook and look at what the rules do in fact say, and note that none of them actually get you what you want.)



    Indeed, the example you give is a good example of the limitations of these rules: while we can often manipulate limits "algebraically," we can't do this in all cases, and some hypotheses are needed. It's also a good example of why proofs are important, since there are plenty of plausible-sounding statements which are in fact false.







    share|cite|improve this answer












    share|cite|improve this answer



    share|cite|improve this answer










    answered 8 hours ago









    Noah SchweberNoah Schweber

    140k10 gold badges168 silver badges318 bronze badges




    140k10 gold badges168 silver badges318 bronze badges














    • $begingroup$
      Hmmm, thank you very much, I didn't know about this caveat! We only scratched the surface of what we can do with limits, as I only studied them in the context of sequences and series. Jesus bless and keep you always. Amen.
      $endgroup$
      – Gregor Perčič
      8 hours ago
















    • $begingroup$
      Hmmm, thank you very much, I didn't know about this caveat! We only scratched the surface of what we can do with limits, as I only studied them in the context of sequences and series. Jesus bless and keep you always. Amen.
      $endgroup$
      – Gregor Perčič
      8 hours ago















    $begingroup$
    Hmmm, thank you very much, I didn't know about this caveat! We only scratched the surface of what we can do with limits, as I only studied them in the context of sequences and series. Jesus bless and keep you always. Amen.
    $endgroup$
    – Gregor Perčič
    8 hours ago




    $begingroup$
    Hmmm, thank you very much, I didn't know about this caveat! We only scratched the surface of what we can do with limits, as I only studied them in the context of sequences and series. Jesus bless and keep you always. Amen.
    $endgroup$
    – Gregor Perčič
    8 hours ago













    2














    $begingroup$

    Your reference page here starts with this introduction (emphasis mine):




    Listed here are a couple of basic limits and the standard limit laws which, when used in conjunction, can find most limits. They are listed for standard, two-sided limits, but they work for all forms of limits. However, note that if a limit is infinite, then the limit does not exist.




    So everywhere below that warning, if there is a statement that a limit must exist, then it's implicit that the limit must not be $infty$.






    share|cite|improve this answer









    $endgroup$



















      2














      $begingroup$

      Your reference page here starts with this introduction (emphasis mine):




      Listed here are a couple of basic limits and the standard limit laws which, when used in conjunction, can find most limits. They are listed for standard, two-sided limits, but they work for all forms of limits. However, note that if a limit is infinite, then the limit does not exist.




      So everywhere below that warning, if there is a statement that a limit must exist, then it's implicit that the limit must not be $infty$.






      share|cite|improve this answer









      $endgroup$

















        2














        2










        2







        $begingroup$

        Your reference page here starts with this introduction (emphasis mine):




        Listed here are a couple of basic limits and the standard limit laws which, when used in conjunction, can find most limits. They are listed for standard, two-sided limits, but they work for all forms of limits. However, note that if a limit is infinite, then the limit does not exist.




        So everywhere below that warning, if there is a statement that a limit must exist, then it's implicit that the limit must not be $infty$.






        share|cite|improve this answer









        $endgroup$



        Your reference page here starts with this introduction (emphasis mine):




        Listed here are a couple of basic limits and the standard limit laws which, when used in conjunction, can find most limits. They are listed for standard, two-sided limits, but they work for all forms of limits. However, note that if a limit is infinite, then the limit does not exist.




        So everywhere below that warning, if there is a statement that a limit must exist, then it's implicit that the limit must not be $infty$.







        share|cite|improve this answer












        share|cite|improve this answer



        share|cite|improve this answer










        answered 8 hours ago









        Chris CulterChris Culter

        22.7k4 gold badges40 silver badges92 bronze badges




        22.7k4 gold badges40 silver badges92 bronze badges































            draft saved

            draft discarded















































            Thanks for contributing an answer to Mathematics Stack Exchange!


            • Please be sure to answer the question. Provide details and share your research!

            But avoid


            • Asking for help, clarification, or responding to other answers.

            • Making statements based on opinion; back them up with references or personal experience.

            Use MathJax to format equations. MathJax reference.


            To learn more, see our tips on writing great answers.




            draft saved


            draft discarded














            StackExchange.ready(
            function ()
            StackExchange.openid.initPostLogin('.new-post-login', 'https%3a%2f%2fmath.stackexchange.com%2fquestions%2f3351115%2fwhy-dont-i-get-the-correct-limit-of-a-sequence-regardless-of-how-i-arrange-it%23new-answer', 'question_page');

            );

            Post as a guest















            Required, but never shown





















































            Required, but never shown














            Required, but never shown












            Required, but never shown







            Required, but never shown

































            Required, but never shown














            Required, but never shown












            Required, but never shown







            Required, but never shown







            Popular posts from this blog

            ParseJSON using SSJSUsing AMPscript with SSJS ActivitiesHow to resubscribe a user in Marketing cloud using SSJS?Pulling Subscriber Status from Lists using SSJSRetrieving Emails using SSJSProblem in updating DE using SSJSUsing SSJS to send single email in Marketing CloudError adding EmailSendDefinition using SSJS

            Кампала Садржај Географија Географија Историја Становништво Привреда Партнерски градови Референце Спољашње везе Мени за навигацију0°11′ СГШ; 32°20′ ИГД / 0.18° СГШ; 32.34° ИГД / 0.18; 32.340°11′ СГШ; 32°20′ ИГД / 0.18° СГШ; 32.34° ИГД / 0.18; 32.34МедијиПодациЗванични веб-сајту

            19. јануар Садржај Догађаји Рођења Смрти Празници и дани сећања Види још Референце Мени за навигацијуу